4. Write the following quadratics as products of two binomials. f(x) = 6x2 + 66x + 60

Answers

Answer 1

Answer:

f(x)=(6x+6)(x+10)

Explanation:

Given the quadratic expression:

[tex]f\mleft(x\mright)=6x^2+66x+60[/tex]

First, we can rewrite it in the form below:

[tex]f\mleft(x\mright)=6x^2+60x+6x+60[/tex]

Next, factor the terms:

[tex]\begin{gathered} f(x)=6x(x+10)+6(x+10) \\ \implies f(x)=(6x+6)(x+10) \end{gathered}[/tex]

Thus, the quadratics as a product of two binomials is:

[tex]f(x)=(6x+6)(x+10)[/tex]


Related Questions

what is the line segment addition to fall the distance of the given line segment

Answers

By looking at the number line we got:

AB =5

BC=5

CD=35

So:

AC = AB+BC = 5+5=10

AD =AB+AC+CD =5+5+35=45

BD = BC+CD = 5+35 =40

Tim buys a torch and a battery
The torch costs eleven times as much as the battery
Tim pays with a £10 note and gets £4.84 change
How much does the battery cost?

Answers

The cost of battery is £0.43.

Given,

In the question:

Tim buys a torch and a battery

The torch costs eleven times as much as the battery and

Tim pays with a £10 note and gets £4.84 change.

To find the how much does the battery cost?

Now, According to the question:

Let cost of battery = £ x

So, Cost of torch = £ 11x

Based on the given conditions :

=> x + 11x = £ 10 - £ 4.84

=> 12x = £ 5.16

=> x = £ 5.16/ 12

=> x = £ 0.43

Hence, The cost of battery is £0.43.

Learn more about Based on Cost at:

https://brainly.com/question/18297478

#SPJ1

27y over 15y in lowest terms

Answers

Given: 27y over 15y

The expression will be as following:

[tex]\frac{27y}{15y}=\text{?}[/tex]

The result of the given expression will be:

[tex]\frac{27y}{15y}=\frac{27}{15}=\frac{3\cdot9}{3\cdot5}=\frac{9}{5}[/tex]

The answer is 9/5

It can be written as a mixed number = 1 4/5

A large glass holds 5/6 cup of water spouse you have 8 cups of water you want to fill both glasses how many of each size glass can you fill

PLEASEE HURRY

Answers

The number of large glasses that will be filled is 10 glasses.

What is a fraction?

A fraction is simply a piece of a whole. The number is represented mathematically as a quotient where the numerator and denominator are split. In a simple fraction, the numerator as well as the denominator are both integers.

Given that the large glass holds 5/6 cup of water and you have 8 cups of water. The number of glasses filled will be:

= 8 ÷ 5/6

= 8 × 6/5

= 48/5

= 9.6

= 10 glasses approximately

Learn more about fractions on:

brainly.com/question/17220365

#SPJ1

How many times larger is 6 × 108 than 3 × 10-4?

Answers

Answer:

About 24.9 or 25 (if rounded to the nearest whole number or tenth).

At a local print shop, 20 copies can be made for $8. At this rate, how how many copies could be made for $34?

Fill out the table of equivalent ratios until you have found the value of x.
Copies Dollars
20 8

xx 34

It would cost $34 to make
copies.

Answers

Answer:4

Step-by-step explanation:

20 times 8 =$160 which is the total price for all copies

$160:34=4.7....$ but you can't make 0.7... of copy, so we don't count it. You can make 4 copies for $34

Circle O shown below has a radius of 25 inches. To the nearest tenth of an inch,determine the length of the arc, x, subtended by an angle of 137°.

Answers

SOLUTION:

Step 1:

In this question, we are given the following:

Step 2:

The details of the solution are as follows:

[tex]Length\text{ of Arc =}\frac{\theta}{360^0}\text{ x 2}\pi\text{ r}[/tex][tex]\begin{gathered} where\text{ }\theta\text{ = 137}^0 \\ radius\text{ = 25 inches} \end{gathered}[/tex][tex]Length\text{ of arc = }\frac{137^0}{360^0}\text{ x 2 x }\pi\text{ x 25}[/tex][tex]Length\text{ of arc =}\frac{137^0}{360^0}\text{ x }\frac{50\pi}{1}[/tex][tex]Length\text{ of Arc = }\frac{21,519.90968}{360}[/tex][tex]Length\text{ of Arc = 59.77752688 inches}[/tex][tex]Length\text{ of Arc }\approx\text{ 59. 8 inches \lparen to the nearest tenth \rparen}[/tex]

Solve the quadratic equation using the quadratic formula or completing the square. Show exact answers in simplified, radical form; no rounded decimals 2x^2-4x+3=0

Answers

The value of x = x = 1 - [tex]\frac{2\sqrt{2i} }{4}[/tex] and x = 1 + [tex]\frac{2\sqrt{2i} }{4}[/tex] by using quadratic formula

Given,

In the question:

The equation is :

2x^2-4x+3=0

To solve by using the quadratic formula :

Now, According to the question;

[tex]2x^2 -4x+3=0[/tex]

Identify the coefficients

a = 2, b = -4 , c = 3

Use the quadratic Formula :

x = (-b±√(b²-4ac))/(2a)

Substitute into values above formula :

x = -(-4) +√(-4)²-4x 2 x 3))/(2x2)  or

x = -(-4) -√(-4)²-4x 2 x 3))/(2x2)

Determine the sign for exponential or radical expressions

x = -(-4) -√(-4)²-4x 2 x 3))/(2x2)

x =  4 -√16-4x 2 x 3))/(2x2)

Calculate the product or quotient

x = 4 -√16-24/4

x =  4 -√-8/4

x = 4 - √-8 x √-1/ 4

x = [tex]\frac{4\sqrt{2^2.2 } }{4}[/tex] x [tex]\sqrt{-1}[/tex]

Simplify the radical expression:

x = [tex]\frac{4 - 2\sqrt{2} . \sqrt{-1} }{4}[/tex]

Rewrite by definition i = [tex]\sqrt{-1}[/tex]

x = [tex]\frac{4-2\sqrt{2i} }{4}[/tex]

x = 1 - [tex]\frac{2\sqrt{2i} }{4}[/tex]

Hence, The value of x = x = 1 - [tex]\frac{2\sqrt{2i} }{4}[/tex] and x = 1 + [tex]\frac{2\sqrt{2i} }{4}[/tex] by using quadratic formula .

Learn more about Quadratic Formula at:

https://brainly.com/question/9300679

#SPJ1


Dada a função y= -3x²+5x+2

Answers

Answer:

y= -4x+2

Step-by-step explanation:

y= -3x²+5x+2

y= -9x+5x+2

y= -4x+2

In the figure below, when the sund angle of elevation is 50 degrees, the tree casts a shadow 80 feet long. which can be used to find the height of the tree?

Answers

A

1) In this picture, we can see clearly a triangle with an adjacent leg to an angle of 50º and an opposite leg (that tree).

2) So the trigonometric ratio to be used is the one that relates both legs:

[tex]\begin{gathered} \tan (50)=\frac{opposite\text{ leg}}{\text{adjacent}} \\ \tan (50)\text{ =}\frac{x}{80} \end{gathered}[/tex]

3) Hence, the answer is A

How to multiply easily big Numbers

Answers

Answer:

Step-by-step explanation:

Multiplying large numbers together is performed by multiplying the ones digit of the bottom number to the entire top number. After the ones place value has finished multiplying, move to the tens place value in the bottom number and multiply that digit by the top number.


I hope it helps!

As Evin is driving her car, she notices that after 1 hour her gas tank has 7.25 gallons left and after 4 hours of driving, it has 3.5 gallons of gas left in it. Assuming the relationship between h, hours, and g, gallons, is linear, write an equation for g in terms of h.

I will mark brainliest to the first person who answers.

Answers

The equation representing the gas left in h hours is g(h) = 8.5 - 1.25h

What is the concept of a linear equation?

When two expressions are connected with the equals sign (=) in a mathematical formula, it expresses the equality of the two expressions. An equation is an algebraic statement that demonstrates two mathematical expressions are equivalent in algebra, and this is how it is most usually used.

A linear equation is written as

y = mx + b

Where (x, y) is the ordered pair and m is the rate of change and b is the y-intercept.

Let x = time in hours

y = amount of gallons left

So, according to the question, 7.25 gallons of gas is left after 1 hour. So,

(x₁, y₁) = (1, 7.25)

3.5 gallons of gas left after 4 hours, so

(x₂, y₂) = (4, 3.5)

Find the slope

m = (y₂ - y₁) / (x₂ - x₁)

= (3.5 - 7.25)/ (4-1)

= -3.75/ 3

= -1.25

Put values of m = -1.25, (x, y) = (1, 7.25) in y= mx+b

So, 7.25 = -1.25 +b

b = 7.25 +1.25

b = 8.5

So, the linear equation for the situation in terms of g and h is

g(h) = -1.25h + 8.5

To know more about linear equations, visit:

brainly.com/question/15184412

#SPJ1

lf had his hourly salary increase from $9.25 per hour to $10.75 per hour, what's the percent increase in Jordan's salary

Answers

Answer:

16.22%

Explanation:

The percent of increase can be calculated as:

[tex]\frac{v_f-v_i}{v_i}\times100[/tex]

Where vf is the final value and vi is the initial value. So, replacing vf by 10.75 and vi by 9.25, we get:

[tex]\frac{10.75-9.25}{9.25}\times100=\frac{1.5}{9.25}\times100=16.22\text{ \%}[/tex]

Therefore, the percent of the increase is 16.22%

What is the slope?

A) 2/1
B)1/2
C)-2/1
D)-1/2

Answers

Answer:

The slope is 1/2, so B is correct.

Answer:B is correct

Step-by-step explanation: it goes up 1=y and right 2=x and y/x

Multiply binomial by polynomials

Answers

In this case the answer is very simple . .

We must apply the distributive property of multiplication.

[tex](d^2+3)\cdot(d^2\text{ + 2d + 1) }[/tex][tex]d^2\cdot d^2+d^2(2d^{})+d^2+3(d^2\text{) + 3(2d) + 3}[/tex][tex]d^4\text{ + }2d^3+d^2+3d^2+6d+3[/tex][tex]d^4+2d^3+4d^2+6d+3^{}[/tex]

That is the solution. .

In the figure, assume that angles that appear to be right angles are right angles.What is the area of the figure?Use 3.14 for π.The approximate area is ___ square units.Use 3.14 for π.

Answers

We can break apart the figure into a half-circle, rectangle, and a triangle.

Shown below:

We find the area of each region and sum it up to find the area of the total figure.

Area of Half Circle

The volume of the area of a half circle is

[tex]A=\frac{\pi r^2}{2}[/tex]

Given, radius (r) = 3, we find the area:

[tex]\begin{gathered} A=\frac{\pi r^2^{}}{2} \\ A=\frac{\pi(3)^2}{2} \\ A=\frac{9\pi}{2} \\ A=\frac{9(3.14)}{2} \\ A=14.13 \end{gathered}[/tex]Area of Rectangle

The area of a rectangle is found by multiplying the length and width.

Given,

Length = 8

Width = 6

The area is:

[tex]\begin{gathered} A=8\times6 \\ A=48 \end{gathered}[/tex]Area of Triangle

The area of a triangle is given by the formula,

[tex]A=\frac{1}{2}bh[/tex]

Where

b is the base length and h is the height

Given,

b = 3

h = 6

The area of the triangle is,

[tex]\begin{gathered} A=\frac{1}{2}bh \\ A=\frac{1}{2}(3)(6) \\ A=9 \end{gathered}[/tex]The total area of the figure >>>

14.13 + 48 + 9

= 71.13 square units

A mixture of orange paint contains 8 tsp of red paint and 12 tsp of yellow paint. To make the same color of orange paint using 18 tsp of red paint, how much yellow paint would you need to use? Consider using the double number line to help
with your thinking.

Answers

After doing some mathematical operations, we know that 27 tsp of yellow colors will be needed with 18 tsp of red paint.

What are mathematical operations?An operation is a function in mathematics that transforms zero or more input values into a clearly defined output value. The operation's arity is determined by the number of operands. The rules that specify the order in which we should solve an expression involving many operations are known as the order of operations. PEMDAS stands for Parentheses, Exponents, Multiplication, Division, Addition Subtraction (from left to right).

So, yellow paint is used in 18 tsp of orange paint:

So,

Tsp of orange: 8 tsp of red + 12 tsp of yellowMake new but the same orange paint: 18 tsp of red + x tsp of yellow

Now, form and solve an equation as follows:

8/12 = 18/x12(18) = 8x8x = 216x = 216/8x = 27

Therefore, after doing some mathematical operations, we know that 27 tsp of yellow colors will be needed with 18 tsp of red paint.

Know more about mathematical operations here:

https://brainly.com/question/28937023

#SPJ1

which value of x below will solve equation 3/4x+10=4

Answers

3/4x+10=4

Then solve for x

3/4x = 4 - 10

x = (4-10)/(3/4)

. = -6/3/4

. = -24/3

. = -8

In consecuence , answer IS

x= -8

Find the sum (3.2+4x)+(18.25+6x)

Answers

Answer:

10x+21.45

Step-by-step explanation:

3.2+4x+18.25+6x

we combine like terms

4x+6x+3.2+18.25

10x+21.45

hopes this helps please mark brainliest

21.45 + 10x

First write the equation without parentheses:

3.2 + 4x + 18.25 + 6x

Then combine like terms:

4x + 6x = 10x,

18.25 + 3.2 = 21.45

Plug the results and rewrite the expression:

21.45 + 10x

Finally, 21.45 + 10x should be your answer.

I’m pretty sure since this is an expression (because there’s no equal sign) and not an equation, you don’t determine the actual value of x or the sum of the numbers. In order to do that, you would need to be provided with the value of x or a number with an equal sign that the problem would equal to, so that’s why it’s still written with a plus sign as an expression.

I hope this helps! Please correct me if I’m wrong! :)

A(6,2), B(2,4) and C(8,-4) are three points in a plane. P is a point on the line BC and Q is a point in the plane such that PAQB is a rectangle. Find the:
(i) equations of the sides AP and BP,
(ii) coordinates of the point P,
(iii) area of the rectangle. ​

Answers

(i) The equations of the sides AP and BP are y = 0.75x - 2.5 and y = (-4/3)x + (20/3), respectively.

(ii) The coordinates of the point P are (4.4, 0.8).

(iii) The area of the rectangle is 8 square units.

A (6, 2), B (2, 4) and C (8, -4) are three points in a plane. P is a point on the line BC, and Q is a point in the plane, such that PAQB is a rectangle.

The point P lies on the line BC. The equation of the line BC will be the same as that of BC. The equation of line BP is calculated below.

(y - y1) = [(y2 - y1)/(x2 - x1)]*(x - x1)

(y - 4) = [(-4 - 4)/(8 - 2)]*(x - 2)

y - 4 = (-8/6)*(x - 2)

y = (-4/3)x + (20/3)

The line AP must be perpendicular to the line BP because they are the sides of a rectangle. The equation for AP is calculated below.

y = mx + c

y = (3/4)x + c

It passes through (6, 2).

2 = (3/4)*6 + c

c = 2 - 4.5

c = -2.5

The equation of AP is y = 0.75x - 2.5.

The coordinates of P are calculated by finding the intersection point of the equations of AP and BP. The coordinates of P are (4.4, 0.8).

The length of the rectangle is BP. The width of the rectangle is AP. These are calculated below using the distance formula.

BP = √[(2 - 4.4)² + (4 - 0.8)²]

BP = √(5.76 + 10.24)

BP = 4

AP = √[(6 - 4.4)² + (2 - 0.8)²]

AP = √(2.56 + 1.44)

AP = 2

The area of the rectangle is calculated below.

A = BP*AP

A = 4×2

A = 8

To learn more about equations, visit :

https://brainly.com/question/10413253

#SPJ9

Which table shows a function that is decreasing only over the interval (-1, infinity)

Answers

The last table shows the function that is decreasing only over the interval [tex](-1, \infty)[/tex].

From given tables we clearly see that in below table, the functions are decreasing as 'x' increases

x f(x)

-3 -5

-1 -1

-1 -1

0 0

1 -4

2 -8

We need to find interval [tex](-1, \infty)[/tex].

In first table,f(x)=-5,-2,-1,2. Here the functions are in increasing order

In second table, f(x)=-7,-6,1,-1. Here the functions does not follow a specific order

in third table, f(x)=-1,2,1,-6, Here also the functions does not follow a specific order.

In fourth table, f(x)=1,0,-4,-8. Here the functions are in decreasing order.

Hence, the fourth table shows the function that deceases over interval [tex](-1, \infty)[/tex].

To learn more about functions refer here

https://brainly.com/question/19866315

#SPJ9

HELP QUICK 30 POINTS!!!!

What equation is graphed in this figure?

A. y−4=−2/3(x+2)


B. y+2=−3/2(x−2)


C. y+1=−2/3(x−3)


D. y−3=3/2(x+1)

Answers

Answer:

B is correct. The slope is -3/2.

Answer:D

Step-by-step explanation:

What is the value of x in the triangle?

a 30-60-90 triangle with long leg length x and shorter leg length of 7 times the square root of 3

Answers

Answer:

sin 30 = 7 *3^1/2 / c

sin 60 = x / c           definition of  30-60-90 -

sin 30 / sin 60 = 7 * 3^1/2 / x = .5 / .866 = .577

x = 7 * 3^1/2 / .577 = 21

Check:       tan 30 = 7 * 3^1/2 / 21 = .577 = tan 30

19. highlight where values show g(x) = 0 and what is the answer20. highlight where values show f(x) > g(x) and what is the answer

Answers

19) g(x)=0 is the value of x at points where the line cuts the x-axis

From the graph that is at:

x=-7, x=1, x=3, and x=9

20) f(x)>g(x) where the curve f(x) is higher than the curve g(x), and this is at:

x<-3, 07

Which of the following shows a graph of a tangent function in the form y = atan(bx − c) + d, such that b equals one half question markgraph of tangent function with one piece that increases from the left in quadrant 3 asymptotic to the line x equals negative 2 times pi passing through the points negative 3 times pi over 2 comma negative 2 and negative pi comma negative 1 and negative pi over 2 comma 0 to the right asymptotic to the line x equals 0 and another piece that increases from the left in quadrant 4 asymptotic to the line x equals 0 passing through the points pi over 2 comma negative 2 and pi comma negative 1 and 3 times pi over 2 comma 0 to the right asymptotic to the line x equals 2 times pigraph of tangent function with one piece that increases from the left in quadrant 3 asymptotic to the line x equals negative 2 times pi passing through the points negative pi comma negative 2 and 0 comma negative 1 and pi comma 0 to the right asymptotic to the line x equals 2 times pigraph of tangent function with one piece that increases from the left in quadrant 3 asymptotic to the line x equals negative 2 times pi passing through the points negative 3 times pi over 2 comma 1 to the right asymptotic to the line x equals negative pi and another piece that increases from the left in quadrant 3 asymptotic to the line x equals negative pi passing through the point negative pi over 2 comma 1 to the right asymptotic to the line x equals 0 and continuing periodicallygraph of tangent function with one piece that increases from the left in quadrant 3 asymptotic to the line x equals negative 7 times pi over 4 passing through the point negative 3 times pi over 2 comma negative 1 to the right asymptotic to the line x equals negative 5 times pi over 4 and another piece that increases from the left in quadrant 3 asymptotic to the line x equals negative 5 times pi over 4 passing through the point negative pi comma negative 1 to the right asymptotic to the line x equals negative 3 times pi over 4 and continuing periodically

Answers

In general, given

[tex]y=a*tan(bx-c)+d[/tex]

b is related to the period of the function, such that, in the case of the tangent function,

[tex]period=\frac{\pi}{b}[/tex]

Remember that the period of tan(x) is equal to pi.

Therefore, in our case,

[tex]\begin{gathered} b=\frac{1}{2} \\ \Rightarrow period=\frac{\pi}{\frac{1}{2}}=2\pi \end{gathered}[/tex]

The period of the function is 2pi, which means that it repeats every 2pi units.

Thus, the answer is the first graph, the one shown below.

Round the number to the place of the underlined digit. 50.633 The rounded number is

Answers

According to the rules of rounding to the nearest numbers, 50.633 rounded to the nearest whole number is equal to 51.

To round 50.633, we have to consider

The tenths’ value of the number 50.633 which is equal to 6 and it is also equal to or more than 5.

Therefore, we have to round up the part of 50.633

50, when increases by 1 results in 51

Here, the decimal point and all the following digits (.633) get removed.

Therefore, 50.633 rounded to the nearest number is equal to 51.

To learn more about rounding to the nearest numbers visit https://brainly.com/question/19031319

#SPJ9

The graph shown shows the supply and demand for a widget. What mappens if the price is set at $25.00? Include in your answer how retailers will react to a $25.00 price point.

Answers

From the graph, if the price increases to $25

The quantity demand will decrease

Quantity supply increases --------This is the reaction from the retailers

Perform the indicated operations and write the result in
standard form.
6) 3√√-16 +2√√-64
A) -28i
C) -28
B) 281
D) 28

Answers

The standard form of the expression  3√-16 +2√-64 is 28i

The square root of minus one √(−1) is the "unit" Imaginary Number, the equivalent of 1 for Real Numbers. In mathematics the symbol for √(−1) is i for imaginary.

We need to write the expression is  3√-16 +2√-64 in standard form

The value of √-1 is i

3√-16 + 2 √-64

3√-1 √16 + 2√-1 √64

3 i 4 + 2 i 8

12 i + 16 i

= 28 i

Therefore, the standard form. of the expression  3√-16 +2√-64 is 28i

To learn more about complex numbers refer here

https://brainly.com/question/10662770

#SPJ9

Jason is a high school basketball player. In a particular game, he made some free
throws (worth one point each) and some two point shots. Jason made a total of 13
shots altogether and scored a total of 20 points. Write a system of equations that
could be used to determine the number of free throws Jason made and the number of
two point shots he made. Define the variables that you use to write the system.

Answers

The total number of free throws shots Jason made was 6 and the total number of two point shots Jason made was 7.

First, let us understand the variables:

Any number, vector, matrix, function, its argument, set, or one of its elements can be specifically represented by a variable.

Let x and y be the quantity of shorts.

x is the number of attempts at the free throw line.

y is the quantity of two-point attempts.

According to the question, we have;

x + 2y = 20 ............equation 1

x + y = 13    ............equation 2

From equation 2 we can take out the value of x;

x = 13 - y     ............equation 3

substituting the value of x in equation 1, we will get;

(13 - y) + 2y = 20

13 - y + 2y = 20

13 + y = 20

y = 20 - 13

y = 7

putting the value of y in equation 3, we will get

x = 13 - 7

x = 6

Thus, the total number of free throws shots Jason made was 6 and the total number of two point shots Jason made was 7.

To learn more about variables visit:

https://brainly.com/question/28248724

#SPJ1

Rita is playing a game by rolling a rectangular prism made up of cubes. The prism is 6 cubes wide, 3 cubes high, and 4 cubes deep. Rita rolls the cube and says that the top view is a 6-unit by 4-unit rectangle.

Which statement explains whether Rita could be correct?
Rita could be correct because the cube could land so the 3-cube height and the 4-cube depth make up the top face of the cube.
Rita could be correct because the cube could land so the 6-cube width and the 4-cube depth make up the top face of the cube.
Rita could not be correct because the top face of the cube is made up of the 6-cube width and the 4-cube depth.
Rita could not be correct because the top face of the cube is made up of the 6-cube width and the 3-cube height.

Answers

Answer:

Step-by-step explanation:

Answer:

Option B: Rita could be correct because the cube could land so the 6-cube width and the 4-cube depth make up the top face of the cube.

Step-by-step explanation:

We are told the dimensions of the prism are;

Width = 6 cubes wide

Height = 3 cubes high

Depth = 4 cubes deep.

Other Questions
Is a triangle with sides that measure 3 inches, 4 inches, and 5 inches a right triangle? Solve the system.x + y + 2z = -1x+ y + 8z = -7(x-9y - 2z = -37 What is the solution to 4x+63 18? proof-of-work is a way to validate transactions based and achieve the distributed consensus. group of answer choices true false Match the figure at the right with the number that represents the sum of the interior angles for that figure. Tommy throws a ball from the balcony of his apartment down to the street. The height of the ball, in meters, is modeled by the function shown in the graph. What's the average rate of change of the height of the ball, in meters per second, while it's in the air?Question options:A) 23B) 23C) 32D) 32 A student is measuring the length of an icicle, y, every hour, x. The icicle is currently 14 inches long and is melting at a rate of 0.9 inches per hour. Find and interpret the slope for this relationship. 0.9; for every additional hour, the length of the icicle decreases by 0.9 inches 0.9; for every additional hour, the length of the icicle increases by 0.9 inches 14; the length of the icicle when the student first measures it 14; the length of the icicle when the student first measures it ( 3y + 1 )( 3y - 1 )Determine each product 2. Find the area: Upload a picture of your work or type it out here 25 cm 123 cm 21 cm Write the expression with a single rational exponent 1/x to the -1 power 2.8 -2 3/4 -31/8 2.2 from least to greatest Use the techniques of College Algebra to show how to write an equation for the quadratic graphed below.x-intercepts: (-3,0) and (1,0). y-intercept: (0,1) The magnetic field at point P is zero. Findthe distance, r2, from P to the second wire,1.I = 4.32 A P------0.831 m1 = 7.50 Ar2r = [?] m on a family trip mr perers travels 130 miles in two hours at this rate how many miles will he travel in 30 minutes? Assignment: 5.4 Bellwork Wednesday 2/03 Problem iD: PRABRBYC Rose wants to construct a fence around her garden. The garden is circular in shape with a diameter of 9 ft. What is the length of fencing material she will need to fence around the outside her garden? ___ H2SO4 + __KOH __K2SO4 +__H2 Obalance the equation System word problems 1. The cost of 3 boxes of envelopes and 4 boxes of paper is $13.25. 2 boxes of envelopes and 6 boxes of paper cost $17. What is the cost of each?Define Variables:System of Equations:Answer as a sentence: Can you please help me fast which relation is a function A.(2,3),(1,5),(2,7)B.(-1,5),(-2,6),(-3,7)C.(11,9),(11,5),(9,3)D.(3,8),(0,8),(3,-2) Which list shows the numbers below in order from least to greatest?5.78, -5.9, 58%, 23-5.9, 23. 5.78, 58%2.-5.9, 58%, 5.78-5.9, 23. 58%, 5.7858%, 23. 5.78, -5,9o